1answer.
Ask question
Login Signup
Ask question
All categories
  • English
  • Mathematics
  • Social Studies
  • Business
  • History
  • Health
  • Geography
  • Biology
  • Physics
  • Chemistry
  • Computers and Technology
  • Arts
  • World Languages
  • Spanish
  • French
  • German
  • Advanced Placement (AP)
  • SAT
  • Medicine
  • Law
  • Engineering
muminat
2 years ago
7

Verify that the Divergence Theorem is true for the vector field F on the region E. F(x, y, z) = z, y, x , E is the solid ball x2

+ y2 + z2 ≤ 64 For your answer, put in the flux across the boundary of E with respect to the outward normal.
Mathematics
1 answer:
Setler [38]2 years ago
8 0

\vec F(x,y,z)=\langle z,y,x\rangle

has divergence 1, so by the divergence theorem, the flux of \vec F across the boundary of E is exactly the volume of E,

\displaystyle\iiint_E\mathrm dV=\int_0^\pi\int_0^{2\pi}\int_0^8\rho^2\sin\varphi\,\mathrm d\rho\,\mathrm d\theta\,\mathrm d\varphi

or simply \dfrac43\pi8^3=\boxed{\frac{2048\pi}3}.

You might be interested in
Deborah bought a computer for 25% off the list price. If the list price is $2,428, how much did Deborah spend? A. $607 B. $1,821
Softa [21]

Answer:

Option A, \$607

Step-by-step explanation:

Given: Deborah bought a computer for 25\% off the list price.

The list price is \$2,428.

To find: The amount Deborah spent on the computer.

Solution: We have,

List price of the computer =\$2,428.

Deborah bought the computer for 25\% off the list price.

The amount Deborah spent on the computer =25\% \times 2,428

=\frac{25}{100}\times2428=\frac{2428}{4} =\$ 607.

So, Deborah spent \$607 on the computer.

Hence, option A is correct.

5 0
2 years ago
PLS ANSWER AND LOOK UNDER THE QUESTION TOO :)))
Sunny_sXe [5.5K]

D. (-2,-1)

Hope it helps you.

7 0
2 years ago
Read 2 more answers
Solve 2x+6 ≤ 10 or 2x + 8 > 20.
Firlakuza [10]

Answer:

<h2>A. x ≤2 or x>6</h2>

Step-by-step explanation:

2x+6 ≤ 10  or  2x + 8 > 20

⇔ 2x ≤ 4  or  2x > 12

⇔ x ≤ 2  or  x > 6

6 0
2 years ago
What is the value of x?<br> A. 52/3<br> B.3/4<br> C.17<br> D. 52/7
MrRa [10]

Answer:

d

Step-by-step explanation:

look at the x and y axis

5 0
3 years ago
One recipe makes 8 1/2 cups of potato salad. If one serving is 1/2 cup, how many servings does the recipe make?
Aleonysh [2.5K]

Answer:

17

Step-by-step explanation:

8.5 / 0.5

7 0
2 years ago
Other questions:
  • Help please!!!!!!!!!!!!!!!
    8·1 answer
  • On Monday, a bicyclist traveled 79.2 miles in 6 hours. On Tuesday, he traveled the same number of miles, but in half the time. W
    12·1 answer
  • What property is 12(x+4)=12x+48
    11·2 answers
  • What is the perimeter inyards of aroom that is 10 yards wide and 18 yards long​
    9·1 answer
  • At aa car dealership, for every 4 silver cars that are sold, 6 red cards are also sold. If 90 cars. Total are sold in a month, h
    7·1 answer
  • PLEASE HELP ME FOR 15 POINTS AND BRAINLIEST QUICK! thanks.
    8·2 answers
  • Can someone dooo ittttttttt
    10·1 answer
  • Vector u has initial point at (3, 9) and terminal point at (–7, 5). Vector v has initial point at (1, –4) and terminal point at
    8·2 answers
  • Find the variable. m angle BDJ=7y+2 , m angle JDR=2y+7
    15·1 answer
  • Square root y is a rational between square root<br><br> 33 and square root 50 find y​
    14·1 answer
Add answer
Login
Not registered? Fast signup
Signup
Login Signup
Ask question!